Do direct limits commute with taking subgroups

abelian-groupscategory-theory

If I have two directed systems of abelian groups $(A_i, \mu_{ij})$ and $(B_i, \mu_{ij}|_{B_i})$ with $B_i \unlhd A_i$ for all $i$ then can I conclude that $\varinjlim B_i$ is isomorphic to a subgroup of $\varinjlim A_i$?


Here's my attempt to reason through that. Recall that $$\varinjlim A_i = \bigoplus_i A_i \bigg/D_A$$ where $D_A$ is the subgroup of $\bigoplus_i A_i$ generated by all elements of the form $x_i – \mu_{ij}(x_i)$ where $i \leq j$ and $x_i \in A_i$.

Note that $\varinjlim B_i$ can't be a true subgroup of $\varinjlim A_i$ because of how the direct limit of a directed system of abelian groups is defined.

Now if we let $D_B$ denote the corresponding subgroup of $\bigoplus_i B_i$ in the definition of $\varinjlim B_i$, then by using the universal property of the direct limit $\varinjlim B_i$ applied to $\varinjlim A_i$ we can find the existence of a (well-defined) homomorphism $\phi : \varinjlim B_i \to \varinjlim A_i$ given by $\phi(b + D_B) = b+D_A$. Now if I can show that $\ker \phi = 0$ then I would be done, but I don't see how I could show that.

Best Answer

If by "directed limit" you mean "filtered colimit" (which I imagine is the case here) then the answer will always be yes, the main reason being that filtered colimits commute with finite limits in $\mathbf{Ab}$. $\require{AMScd}\def\id{\operatorname{id}}$

If $A:J\to\mathbf{Ab}$ is a filtered diagram (i.e., directed system) and $B$ is a subdiagram (this is just my way of phrasing what you describe in your question), then like you mentioned the inclusion $i:B\Rightarrow A$ induces a morphism $i:\varinjlim B\to\varinjlim A$ of colimits.

Now, this is a monomorphism (i.e., injective) iff \begin{CD} \varinjlim B @>\id>> \varinjlim B \\ @V\id VV @VViV \\ \varinjlim B @>>i> \varinjlim A \end{CD} is a pullback square, which is true because this is the colimit of the system of pullback squares \begin{CD} B_j @>\id>> B_j \\ @V\id VV @VV\subseteq V \\ B_j @>>\subseteq> A_j \end{CD} for $j\in J$.


If "directed limit" just meant an arbitrary colimit, the answer is no: for example, $\Bbb Z$ is the colimit of the diagram $\Bbb Z\xleftarrow{\id}\Bbb Z\xrightarrow{\id}\Bbb Z$, but if we look at the subdiagram $\Bbb Z\leftarrow0\rightarrow\Bbb Z$ the colimit is $\Bbb Z\oplus\Bbb Z$, and the induced map $\Bbb Z\oplus\Bbb Z\to\Bbb Z$ is given by $(n,m)\mapsto n+m$, which is certainly not injective (and $\Bbb Z\oplus\Bbb Z$ simply cannot be a subgroup of $\Bbb Z$).


To see this more concretely, the direct limit $\varinjlim A$ as the result of putting all the elements of every $A_j$ together, identifying elements via the transition maps. If the diagram is not filtered, then it is possible that "the reason $a_j\in A_j$ and $a_{j'}\in A_{j'}$ sum to zero is that they both come from some $A_k$ with $k\leq j,j'$ and in there, they sum to zero" so if your subgroup $B_k$ does not include $a_j$ and $a_{j'}$, then these elements may not sum to zero in $\varinjlim B$.

However, with a filtered diagram, any $j,j'$ have an $\ell\geq j,j'$, meaning for any $a_j\in A_j$ and $a_{j'}\in A_{j'}$ that we can find these elements in $A_\ell$ and perform the addition there. Therefore, if $a_j+a_{j'}=0$, this fact will be witnessed in some group in the directed system containing both of those elements. This resolves the original issue because if $a_j\in B_j$ and $a_{j'}\in B_{j'}$ as well, then they must both lie in $B_\ell\subseteq A_\ell$ and therefore they will still sum to zero.

In other words, an element of $\varinjlim A$ is zero precisely when it is the image of the zero in one of the $A_{j'}\to\varinjlim A$. If $\phi:\varinjlim B\to\varinjlim A$ sends some $b$ to zero, note that $b$ comes from some $B_j$, and $\phi(b)$ is the image of some zero in $A_{j'}$. Take $\ell\geq j,j'$, then $b$ gets sent via the transition map $B_j\to B_\ell$ to the same element that $0\in A_{j'}$ is sent to via its transition map (which is again $0$). That is, $b$ vanishes in $B_\ell$, and therefore $b$ must have already been zero in $\varinjlim B$, showing that $\phi$ is injective.

Related Question